someone help me please, this topic is based on slopes.

Someone Help Me Please, This Topic Is Based On Slopes.

Answers

Answer 1

Answer:

 run / rise =5/1

Step-by-step explanation:

The run is 5 and the rise is 1


Related Questions

) Which statements about -3 and -7 are true? Choose ALL the correct answers.
-3 is less than -7.
-3 is greater than -7.
The absolute value of -3 is less than the absolute value of -7.
The absolute value of -3 is equal to the absolute value of -7.
.) The absolute value of -3 is greater than the absolute value of -7.
DONE

Answers

Answer: -3 is greater than -7 and the absolute value of -3 is less than the absolute value of -7.

Step-by-step explanation:

For NEGATIVE numbers, the closer the value is to 0, the greater it is. -3 is closer to 0 than -7 is, so -3 is greater than -7. For POSITIVE numbers, the further away the number is from 0, the greater it is. The absolute value of -3 is 3 and the absolute value of -7 is 7, and 7 is farther away from 0 than 3 is. So, the absolute value of -3 is less than the absolute value of -7. Hope this helps!

Can someone please help me with this proof?!

Given: AC and BD bis, each other.

Prove: ABCD is a parallelogram

Answers

Answer:

because AC and BD bis => AX = XC; BX = XD

ΔAXD ≅ ΔCXB (SAS) because: AX = CX

                                                    DX = BX

                                                    m∠AXD = m∠BXC ( 2 opposing angles)

because ΔAXD ≅ ΔCXB (SAS)

=> AD = BC and m∠DAX = m∠BCX

because m∠DAX = m∠BCX => AD//BC

ABCD has AD = BC and AD//BC => ABCD is a parallelogram

Step-by-step explanation:

how many solution does -3 (4 - 3x) = 9x - 12

Answers

Answer: infinite solutions

Step-by-step explanation:

(−3) (4) + (−3) (−3x) = 9x + −12

−12 + 9x = 9x + −12

9x − 12 = 9x − 12

9x − 12 − 9x = 9x − 12 − 9x

−12 = −12

−12 + 12 = −12 + 12

0 = 0

If y varies directly as x, and x = 13 when y = 6, what is y when x = 65?

Answers

Answer:

y=30

Step-by-step explanation:

what kinda math is this​

Answers

Shapeeee and graph and line

which
among the following is a quadratic equation?
A. (x + 1 )² = x² – 1
B. (x + 1 ) (x – 2 ) = x – 3
C. x (x + 1 )²= 0
D. 2x² = 2 ( x – 1)²
​with explanation pls...

Answers

B.

Because when doing the math, letter B fits the criteria of a quadratic equation. Which is ax^2 + bx + c = 0.

A is wrong because if we were to do the math, there would be no ax^2. The most important thing in a quadratic equation is the term ax^2, bx and c do not matter as much.

C is wrong because there cannot be a greater exponent than ^2.

D is wrong because of the same reason as letter a.

30 pts for this one.... (only if you give a reasonable answer no BS)

Use the scenario below to answer questions 2 - 4

A Tech Software salesperson earns a base salary of $40,000 plus a commission of $3000 for every subscription to the software they sale.

2. Write an equation, in slope-intercept form, that shows the total amount of income the salesperson earns if he sells x subscriptions in a year.

3. What does the y-intercept of the equation represent in the context of the problem?
a) the base salary of the salesperson
b) the total salary of the salesperson per year
c) the amount of money the salesperson makes per software sale.
d) none of these

4. What does the slope of the equation represent in the context of the problem?
a) the commission per subscription
b) the total amount of money the salesperson makes
c) a one time, yearly commission for all the subscriptions
d) the base salary of the sales person


WANT A EXTRA 10 PTS?

comment on this and ill make another problem for 10 pts!

Answers

Answer:

y = 3000x + 40000a) the base salary of the salespersona) the commission per subscription

Step-by-step explanation:

Slope intercept form is found by [ y = mx + b ] where m = slope and b = y-intercept. We are given that a salesperson gets $40000 as a base salary. Since this value doesn't change it is the starting value, in other words the y-intercept. The salesperson also gets $3000 for every subscription they sell. Since this amount grows at a constant rate, that is the slope. Now, we can write the slope intercept form of [ y = 3000x + 40000 ].

Best of Luck!

Answer:

1. y = 3000x + 40000

2. Option A: base salary of the salesperson

3. Option A: the commission per subscription

Step-by-step explanation:

1. The commission of $3000 will be the first number, then will come the salary. Now, since it has to be in slope form, it is on the y-intercept, so we write y.

2. It is the base salary because it is the only number that does not have a missing value, 3000x does.

3. The commission per subscription is the answer because the context of the whole slope is the commission per subscription. Question 2 was the y-intercept.

1.
Susan has five blocks of candy. She wantsstwo ciwe chook into -size pieces of
candy. How many pieces of candy willstrebe anettonale?

Answers

B) I can’t read this.
This is the weirdest thing I’ve ever seen

1)
Bill and John competed in the “Shot Put" event in their school's track
meet. Bill threw the shot 21 feet and 6 inches, while John threw the
shot 17 feet and 4 inches. How much greater was Bill's throw than
John's?

Answers

I think 4ft 2in or 50in but im not 100% sure

Find the value of x for which m || n.

Answers

Answer:

40

Step-by-step explanation:

The angles are equal to eachother so 4x-26 = 3x+14

40 it’s really easy just add 20+20 - 4 =

What is the equation of the line through the origin and (3, 4)?
O A. y = 2
O B. y = x
O
C. y = 4x
O D. y = 3x

Answers

Answer:

A logarithm is the power to which a number must be raised in order to get some other number (see Section 3 of this Math Review for more about exponents). For example, the base ten logarithm of 100 is 2, because ten raised to the power of two is 100: log 100 = 2. because.

STep-by-step                                            Ihave a felling its b

please help me i help you i send a picture

Answers

Answer:

30

Step-by-step explanation:

ita directly proportional so its 30.

this is because 20 times 2 is 40 and 15 times 2 is 30

Find the slope of the line through the pair of points.
(17,-7) and (13,-4)]

Answers

Answer:

3/4

Step-by-step explanation:

(17,-7) And (13,-4).  First take -4 and subtract it from-7.  When you do that you get 3.  Then Subtract 13-17.  You get 4.  Write 3 and 4 as a fraction.  3/4.  The 3 is the x-value and the 4 is the y value. So 3/4 is your slope

( BRAINLIEST IF CORRECT)If your business is generating $500 per month in cash flow, and you are trying to save up for a new pitching machine that costs $4,300 , about how long will it take you to save up?
6months
7months
9months
11months

Answers

Answer:

9 months

Step-by-step explanation:

got it right

If (2,-5) is a point on a line with an
undefined slope, which of these is
another point on the line?
A (0,-5)
B (-5,2)
C(2,3)
D(0,3)

Answers

Answer:

(2, 3)

Step-by-step explanation:

lines with undefined slopes are vertical lines

25p2 - 144
need to factor​

Answers

Answer: (5p+12)(5p-12)

Step-by-step explanation:

Factor 25p2−144

25p2−144

=(5p+12)(5p−12)

Answer:

(5p+12)(5p−12)

To check:

(5p+12)(5p−12)

=(5p+12)(5p+−12)

=(5p)(5p) +(5p)(−12)+(12)(5p)+(12)(−12)

=25p2−60p+60p−144

=25p2−144

For each problem, explain or show your reasoning, a. 160 is what percentage of 40? b. 40 is 160% of what number? c. What number is 40% of 1602​

Answers

Answer:

A. 160 Out of 40 is 400%./// B. The answer is 25/// C.  The answer is 3905%

Step-by-step explanation:

( A )  One step solution:

You can easily find the answer in one step, just multiplying the part by 100 then dividing the result by the percentage.

So, 160 is 40% of =  160 X 100     = 400

                                        40

                                             

( B ) The same thing as A but the numbers are switched

       One step solution:

You can easily find the answer in one step, just multiplying the part by 100 then dividing the result by the percentage.

So, 40 is 160% of = 40 × 100      = 25

                                          160

( C ) Working out a percentage increase or decrease between two numbers is pretty simple. The resulting number (the second input) is 1602 and what we need to do first is subtract the old number, 40, from it:

1602 - 40 = 1562

Once we've done that we need to divide the result, 1562, by the original number, 40. We do this because we need to compare the difference between the new number and the original:

1562      = 39.05          

40

We now have our answer in decimal format. How do we get this into percentage format? Multiply 39.05 by 100?

39.05 x 100 = 3905%

160 is percentage of 40,  40 is 160% , number is 40% of 1602​ are 64 , 64 and 640.8 respectively.

What is a percentage?

A ratio or value that may be stated as a fraction of 100 is called a percentage. And it is represented by the symbol '%'.

Problem A,

40% of 160 = 160 x 40 / 100

                  = 64

Problem B,

160% of 40 = 160 x 40 / 100

                  = 64

Problem C,

40% of 1602 = 1602 x 40 / 100

                     = 640.8

Therefore, 160 is percentage of 40,  40 is 160% , number is 40% of 1602​ are 64 , 64 and 640.8 respectively.

To learn more about the percentage;

https://brainly.com/question/24159063

#SPJ2

Find the slope from the points given:
(18, -5), (18, 20)

Answers

Answer:

UNDEFINED

Step-by-step explanation:

change in y over the change in x is how you'd find slope, however, x is the same in both of these, making this a limitation not an equation, therefore the slope is undefined 18-18 = 0 -> Anything divided by zero as a slope is undefined.

If the perimeter of a triangle is 7x + 13, and two sides measure -6x + 12 and 5x – 5. Find the missing side.

Answers

Answer:

6x+20

Step-by-step explanation:

7x+13= (-6x+12) + (5x-5) + y

7x+13= -1x+ 7 + y

6x+20=y

I hope im right!

What is the dollar value of 10 pennies and 3 nickels? What is the dollar value of p
pennies and n nickels?

Answers

Answer:

25

Step-by-step explanation:

Jack wants to buy a piggy bank at 6000 dollars . Black Friday is coming at November 6th and there are 80%of all goods how much will he now pay ?​

Answers

Jack will pay $1,200

Answer: 1,200 dollars

Step-by-step explanation:

We know that the cost before the discount is 6,000.

The discount price is 80% off.

So, subtract 4,800 by 6,000 because 80% of 6,000 is 4,800.

Now, we have to add the two zeros. We get 1200.

It is the answer to the question.

Dylen was running for president of his class. He wanted to make a campaign flier for each student in his school. If there are about 2,995 students in Dylen’s school,what is a reasonable amount of fliers for Dylen to print out? Round your answer and show your thinking.

Answers

I’d say about 3000 Bc you need to round up

help this is my last anmepent plz get help

Answers

Answer:

6 to the power of 10

Step-by-step explanation:

Since there are parenthesis, you multiply the exponents instead of adding. 5*2=10

What is the correct answer? ​

Answers

The answer is Cccccccccccccccccccccc

Answer:

C)  y = 10/3x - 5

Step-by-step explanation:

Slope intercept equation is y = mx + b:

    m is the slope

    b is y-intercept

When you read slope you can also read it as rise over run. Rise is move up or down on the y axis depending if number is negative or positive. Run is move left or right on x axis depending if number is negative or positive.

Example: y = [tex]\frac{10}{3} x - 5[/tex]

Start at the y-intercept which is -5. On graph you can see the line crosses the y axis at (0, -5). From this point rise 10 then run 3 you'll end up at point (3, 5). Connect the dots of the two points and you'll see a line like the one in the problem.

What is the answer to this x + y = 16

Answers

x + y = 16

y = -3x

x=-8

y=24

-8+24=16

16=16

-3(-8)=24

24=24

i dont really get the second and third ones please help asap :/

Answers

Answer:

Step-by-step explanation:

1) y = 5x

x      y

2     10

3      15

7       5 * 7 = 35

1       5 * 1  =  5

2) a = 4b

a               b

12             3

20            5

4*10=40   10

1                 1/4

3) m = n + 2

m                n

5                  3

10                 8

20                18

1                    -1

Slope intercept and all that stuff

Answers

Answer:

what?

Step-by-step explanation:

If this helps, the slope intercept is y=mx+b

Anyone can help me with this problem

Answers

Answer:

R=50 degrees

Step-by-step explanation:

All of the angle measures together are 180 degrees. You need to make an equation in order to solve this problem:

180= (3x+5) + (11x-1) + (5x+5)

You then combine like terms:

180= (3x+11x+5x) + (5-1+5)

180= 19x+9

Then, subtract 9 from 180:

171= 19x

Then, divide 171 by 19:

9= x

Finally, you plug the value of x into the equation representing angle R:

5 x 9 + 5

45 + 5

50

Find the value of Angle 4.
Find the value of x.

Answers

♥️♥️♥️♥️♥️♥️♥️♥️♥️♥️♥️♥️♥️♥️

a )

[tex]angle \: 4 = 62°[/tex]

♥️♥️♥️♥️♥️♥️♥️♥️♥️♥️♥️♥️♥️♥️

b )

[tex]8x + 6 = 62[/tex]

Subtract sides 6

[tex]8x + 6 - 6 = 62 - 6[/tex]

[tex]8x = 56[/tex]

Divide sides by 8

[tex] \frac{8x}{8} = \frac{56}{8} \\ [/tex]

[tex]x = 7[/tex]

♥️♥️♥️♥️♥️♥️♥️♥️♥️♥️♥️♥️♥️♥️

Answer:

4 = 15.5 degrees

Explanation:

180 degrees - 118 degrees = 62 degrees

62/4= 15.5 degrees

Answer:

x = 7 degrees

Explanation:

8x+6=62  

62-6=56

8x=56

x=7

The SI unit for measuring mass is the

Answers

Answer:

The SI unit for measuring mass is the kilogram

Other Questions
write and equation of the line that passes through the point (4, -2) and has a slope of -1/2. Help This is easy but I am DUMB you are working on creating a business document with two other co-workers. Based on just information, which of the following pre-writing strategies would be the best for you?(A.P.E.X.) In Selection 2, the author has Rhetta compare Ida to Jane Eyre in order toexplain a major theme.introduce a snobbish tone.create a sense of suspense.clarify a common misconception. What is the value of 2 in 8.702 Which phrases describe a metamorphic rock Students in a sixth-grade class are raising money for an end-of-year camping trip. So far, they have raised $240. This is 2/5 (two-fifths) of the cost of the trip. How much does the trip cost? Help meSolve 5n = 35. Which of the following best describes why Eastern European countries tend to be less developed than Western European countries? A Eastern European countries have lower populations and must rely on immigrants to produce goodsEastern European countries have lower populations and must rely on immigrants to produce goods B Western European countries were more influenced by the Soviet Union after World War IIWestern European countries were more influenced by the Soviet Union after World War II C Eastern European countries were more influenced by a Communist economic systemEastern European countries were more influenced by a Communist economic system D Eastern European countries rely on supply and demand driven market economies Which colonial group was established around 1770 in order to help colonist participate in boycotting against British goods what is the equation in slope intercept form of this graph A man sold an article for $90, thus making a profit of 12.5%. What was the cost price of the article? A linear function contains the following points. What are the slope and y-intercept of this function? Please help me with this is really important here is the picture Help. Dont know what to do John went home what is the subject Read the following scenario and answer the question below.The president has just vetoed a controversial budget bill passed by Congress last week.Which role of the president is represented in the scenario?A.Head of stateB.Cheif diplomatC.Commander in chiefD.Head of the executive branchNEED HELP ASAP I WILL GIVE BRAINLEST what is the rhyme scheme of the poem below?Closed Doors By: Jane Kirby I cry myself to sleep at night Just wishing it would stop. Maybe tomorrow it will cease. Then again, maybe not. You push me and you poke me. You call me hurtful names. To me, it's like a prison. To you, it's fun and games. Your heart is made of ice, But it doesn't even deserve that. Mine was once a rainbow, But you have molded it to the darkest black. I hate you; I hate everyone. There is no one I can trust. Are you proud of what you've done? You've turned my soul into dust. I wish there were a door I could open, A simple path I could tread. I don't want to be stuck here forever In this endless spiral of dread. !!PLEASE HELP!!Written responses for A-B on the causes of WWIICauses of World War II:A. Post-World War IB. AggressionsC. Appeasement plz help me Rachel is five years younger than Monica. If the sum of their ages is 21, find the age of each girl.plz, answer this is due today !!!